If BE−→− bisects ∠ABD and m∠ABD = 66°, find m∠ABE.

Answers

Answer 1

Answer:

∠ABE = 33°

Step-by-step explanation:

Angle bisector:

        Angle bisector is a straight line that divides the angle into two congruent (equal) angles.

So, BE bisects the ∠ABD into two equal angles.

⇒∠ABE will be half of ∠ABD

∠ABD = ∠ABE + ∠BED

66°     = ∠ABE + ∠ABE

66      = 2∠ABE

∠ABE = 66/2

∠ABE = 33°


Related Questions

Which of these is a zero of the polynomials p(y) = 3y^3 - 16 y - 8? * - 8 0 2 - 2

Answers

The zero of polynomials is -2. The polynomials is p(y) = [tex]3y^{3} - 16 y - 8[/tex].

According to the question,

The polynomials is p(y) = [tex]3y^{3} - 16 y - 8[/tex]. In order to find the zero of polynomials only if we substitute the value and polynomials become zero.

p(-2) = [tex]3(-2)^{3} - 16 (-2) - 8[/tex]

= 3(8) + 32 -8

= 0

Hence, the zero of polynomials is -2. The polynomials is p(y) = [tex]3y^{3} - 16 y - 8[/tex].

Learn more about polynomials here

https://brainly.com/question/1720316

#SPJ4

Please help with this

Answers

Answer: Circle lines

Step-by-step explanation:

Hopefully the attached image helps, it is a diagram of all the labeled lines on a circle excluding the radius (BG or GE)

Graph the image of this triangle after a dilation with a scale factor of 2 centered at the origin. Use the polygon tool to graph the triangle.

Answers

The triangle is illustrated below.

How to explain the triangle?

The first thing you should find are the new vertices:

(x, y) ---> (2x, 2y) ---> (x', y')

(0, 0) ---> (2 (0), 2 (0)) ---> (0, 0)

(-4, 4) ---> (2 (-4), 2 (4)) ---> (-8, 8)

(-4, -2) ---> (2 (-4), 2 (-2)) ---> (-8, -4)

Then, you must join the ordered pairs and graph the new triangle.

See the attached graph.

Learn more about triangles on:

https://brainly.com/question/1058720

#SPJ1

You buy a car for $25,000. It depreciates at the rate of 23% per year.
a. Write an exponential equation to model the value of the car.
What will the car be worth in 10 years?
round to nearest cent

Answers

The worth of the car in 10 years is $1,831.67 using an exponential equation approach.

What is an exponential equation?

An exponential equation is the one with exponents such X^3(the 3 is the exponent)

The exponential equation required here is the one where the future value would be lower than current value because the car reduces in value year-in-year-out.

FV=PV*(1-r)^N

FV=future worth of the car

PV=today's value=$25,000

r=depreciation rate=-23%

N=number of years=10

The fact that r is negative means the car is depreciating not appreciating.

FV=$25,000*(1-23%)^10

FV=$1,831.67

Find further explanation on exponential equation below in the link:

https://brainly.com/question/11832081

#SPJ1

An auditorium with 30 rows of seats has 10 seats in the first row. Each successive row has one more seat than the previous row. If students taking an exam are permitted to sit in any row, but not next to another student in that row, what is the maximum number of students that can be seated for an exam

Answers

Answer:

Step-by-step explanation:

380 rows in total

bcs

use sense

PLEASE HELP! Solve each of the following by using the general quadratic formula.

Answers

Using the quadratic formula, the solutions are:

a) [tex]x = \frac{3 \pm \sqrt{41}}{4}[/tex]

b) [tex]x = 1 \pm 2i[/tex]

What is a quadratic function?

A quadratic function is given according to the following rule:

[tex]y = ax^2 + bx + c[/tex]

The solutions are:

[tex]x_1 = \frac{-b + \sqrt{\Delta}}{2a}[/tex]

[tex]x_2 = \frac{-b - \sqrt{\Delta}}{2a}[/tex]

In which:

[tex]\Delta = b^2 - 4ac[/tex]

Item a:

The coefficients are a = 2, b = -3, c = -4, hence:

[tex]\Delta = (-3)^2 - 4(2)(-4) = 41[/tex][tex]x_1 = \frac{3 + \sqrt{41}}{4}[/tex][tex]x_2 = \frac{3 - \sqrt{41}}{4}[/tex]

Item b:

The coefficients are a = 1, b = 2, c = 2, hence:

[tex]\Delta = (2)^2 - 4(1)(2) = -4[/tex][tex]x_1 = \frac{2 + \sqrt{-4}}{2} = 1 + 2i[/tex][tex]x_2 = \frac{2 - \sqrt{-4}}{2} = 1 - 2i[/tex]

More can be learned about quadratic equations at https://brainly.com/question/24737967

#SPJ1

Are the triangles similar? If so, state the similarity and the postulate or theorem that justifies your answer.

Answers

Answer: The triangles are not similar.

Step-by-step explanation:

Two figures are similar simply if they have the same shape, but not necessarily the same size. In a more mathematical sense, similar figures have the same angle measures and proportionate side lengths.

For reference, side lengths are proportionate when the ratios between corresponding/matching sides are the same.

Since we do not have any angles, we will try using the SSS Similarity Theorem, which states that if all three sides of both triangles are in proportion, then the triangles are similar. We will first list all the side lengths and match corresponding ones.

[tex]\triangle ABC \rightarrow 18,20,32\\ \triangle UVT \rightarrow 14,15,24[/tex]

Since we matched corresponding sides, we can now check whether they have the same ratio).

[tex]\frac{18}{14}=\frac{20}{15}=\frac{32}{24}[/tex]

[tex]\frac{9}{7}=\frac{4}{3}=\frac{4}{3}[/tex]

Not all of the side lengths have the same ratio, so these triangles aren't similar. you must multiply 14 by 9/7 to get to 18, but you need to multiply the other two sides by 4/3 to get to their

3x-2y=10 find the y intercept and the x intercept

Answers

Answer:

(0, -5)      (10/3, 0)

Step-by-step explanation:

First, convert the given equation into the slope-intercept form, y = mx + b, where m represents the slope and b represents the y-intercept.

3x - 2y = 10

2y = 3x - 10

y = 3/2x - 5

The y-intercept is -5.

Next, to find the x-intercept, let y = 0 and solve for x. X-intercept is the point where the graph intersects the x-axis, so the y-coordinate of x-intercepts are always 0.

3x - 2*0 = 10

3x - 0 = 10

3x = 10

x = 10/3

X-INTERCEPT

Plug y=0 into the equation and solve the resulting equation 3x=10 for x.

The x-intercept:

[tex]\left(\frac{10}{3},0\right)\approx \left(3.33333333333333,0\right)[/tex]

Y-INTERCEPT

Plug x=0 into the equation and solve the resulting equation −2y=10 for y.

The y-intercept:

[tex]\left(0, -5\right)[/tex]

-6-5
(-5-4)
(-2,2)
2-
4
--2-1₁
4-
2
4 9
(0, -3)
X
What is the equation of the line that is parallel to the
given line and passes through the point (-2, 2)?
Oy=x+4
Oy = x + ¹/2
Oy = -5x + 4
Oy=-5x + ¹2

Answers

The equation of the line that is parallel to the given line and passes through the point (-2,2) is:

[tex]y = \frac{x}{5} + \frac{12}{5}[/tex]

What is a linear function?

A linear function is modeled by:

y = mx + b

In which:

m is the slope, which is the rate of change, that is, by how much y changes when x changes by 1.b is the y-intercept, which is the value of y when x = 0, and can also be interpreted as the initial value of the function.

When two lines are parallel, they have the same slope. In this problem, the given line passes through (-5,-4) and (0,-3), hence the slope is:

m = (-3 - (-4))/(0 - (-5)) = 1/5.

Hence the equation is:

[tex]y = \frac{x}{5} + b[/tex]

When x = -2, y = 2, then:

[tex]y = \frac{x}{5} + b[/tex]

[tex]2 = \frac{-2}{5} + b[/tex]

[tex]b = \frac{12}{5}[/tex]

Hence:

[tex]y = \frac{x}{5} + \frac{12}{5}[/tex]

More can be learned about linear equations at https://brainly.com/question/24808124

#SPJ1

Given the following results from a spinner game after spinning 20 times.
What is the experimental probability of landing on red?

Answers

The experimental probability of landing on red is 7/20 or 0.35 or 35%

How to determine the experimental probability of landing on red?

The complete question is added as an attachment

From the question, we have the following outcomes:

Red = 7

Green = 5

Blue = 5

Yellow = 3

The total number of spins is

Total = Red + Green + Blue + Yellow

This gives

Total = 7 + 5 + 5 + 3

Evaluate the sum

Total = 20

The experimental probability of landing on red is the calculated as:

P(Red) = Red/Total

Substitute the known values in the above equation

P(Red) = 7/20

Evaluate

P(Red) = 0.35

Express as percentage

P(Red) = 35%

Hence, the experimental probability of landing on red is 7/20 or 0.35 or 35%

Read more about probability at:

https://brainly.com/question/25870256

#SPJ1

Help help help help help just need answer

Answers

Answer:

25 miles

Step-by-step explanation:

2 inches represents 10 miles ( divide both parts by 2 )

1 inch represents 5 miles , then

5 inches represents 5 × 5 = 25 miles

Vector u has initial points at (21,12) and it’s terminal point at (19,-8). Vector v has a direction opposite that of u. Who’s magnitude is five times the magnitude of v. Which is the correct form of vector v expressed as a linear combination of the unit vectors i and j?

Answers

The correct form of vector v expressed as a linear combination of the unit vectors i and j is [tex]\vec v = 10\,\hat{i} + 100\,\hat{j}[/tex].

What is the value of a vector with respect to another vector?

First, we need to determine the value of the vector u by subtracting two vectors whose initial points are at the origin:

[tex]\vec u = (19\,\hat{i} - 8\,\hat{j}) - (21\,\hat{i} + 12\,\hat{j})[/tex]

[tex]\vec u = - 2\,\hat{i} - 20\,\hat{j}[/tex]     (1)

According to the statement, vector v is antiparallel to vector u and its magnitude is five times as the magnitude of vector v, which means that (1) must be multiplied by two scalars:

[tex]\vec v = - 1 \,\cdot \, 5\cdot \vec u[/tex]      (2)

Please notice that antiparallelism is represented by the scalar - 1, whereas the dilation is represented by the scalar 5.

[tex]\vec v = 10\,\hat{i} + 100\,\hat{j}[/tex]

The correct form of vector v expressed as a linear combination of the unit vectors i and j is [tex]\vec v = 10\,\hat{i} + 100\,\hat{j}[/tex].

Remark

The statement presents typing mistakes, correct form is shown below:

Vector u has initial points at (21, 12) and its terminal point at (19, - 8). Vector v has a direction opposite that of u, whose magnitud is five times the magnitud of v. Which is the correct form of vector v expressed as a linear combination of the unit vectors i and j?

To learn more on vectors: https://brainly.com/question/13322477

#SPJ1

What is the explicit formula for this sequence?
-9,-3,3,9,15

Answers

Answer: A

Step-by-step explanation:

Normally in choice questions, I would choose to substitute the values into the formulas but I think it is better if I explain to you.

In a sequence, if the numbers increase by d each time and the first number in the sequence is [tex]a_{1}[/tex], then the explicit formula is [tex]a_{1}[/tex] + (n - 1)d when n is the order of the number in the sequence, like n = 1 for the first number, n = 2 for the second number and so on. Using this, we get that the answer to this problem is [tex]a_{n}[/tex] = -9 + (n - 1)6 so the answer is A

Hat is the area of the shaded face of the cylinder is 22m give your answer to the nearest whole number and give the correct units

Answers

The area of the shaded face of the cylinder is 1,520 mm².

How to find the area of the shaded area?

We can see that the shaded area of the cylinder is circular in shape.

This means that we can find the area of the shaded area by finding the area of the circle.

The radius of the circle is given as 22 mm.

The formula for finding the area of a circle is given as:

Area = πr²

= 3.14 × 22 × 22

= 1,519.76

≈ 1520 mm²

Therefore, we have found the area of the shaded face of the cylinder to be 1,520 mm².

Learn more about the area of a circle here: https://brainly.com/question/14068861

#SPJ4

Disclaimer: The question was incomplete, the complete question is attached below.

Causes of variation that can be identified and eliminated are called what?

Answers

The causes of variation that can be identified and eliminated are called; Assignable Causes.

What are the causes of Variation?

There are two primary causes of variation in the quality of a product or process. These two primary causes are called;

Common causes.Assignable causes.

Now, Common causes of variation are defined as random causes that we cannot identify. However, Assignable causes of variation are those that can be identified and eliminated.

Read more about Variation Causes at; https://brainly.com/question/14926046

#SPJ1

Which represents the solution set to the inequality 5.1(3 2.2x) > –14.25 – 6(1.7x 4)? x < –2.5 x > 2.5 (–2.5, [infinity]) (–[infinity], 2.5)

Answers

The solution set to the inequality 5.1(3 + 2.2x) > –14.25 – 6(1.7x + 4) gives x > -2.5

What is an equation?

An equation is an expression that shows the relationship between two numbers and variables.

An independent variable is a variable that does not depend on any other variable for its value whereas a dependent variable is a variable that depend on any other variable for its value.

Inequality shows the non equal comparison of two or more numbers and variables.

Given the inequality:

5.1(3 + 2.2x) > –14.25 – 6(1.7x + 4)

Simplifying:

15.3 + 11.22x > –14.25 – 10.2x - 24

21.42x > -53.55

x > -2.5

The solution set to the inequality 5.1(3 + 2.2x) > –14.25 – 6(1.7x + 4) gives x > -2.5

Find out more on equation at: https://brainly.com/question/2972832

#SPJ1

A math club is researching a golf tournament fund-raiser. it will cost $1,000 to host the tournament. if it rains, the club will lose the investment. if it is sunny, it is expected that the club will collect $4,500 from the participants. if the chance of rain is 20%, what is the expected value for the tournament?

Answers

The expected value of tournament is $2600.

According to the statement

we have given that the tournament will cost $1000 and if it is sunny, it is expected that the club will collect $4,500 from the participants. And we have to find the expected value of a tournament.

So, we know that the

Cost of hosting the tournament : $1,000

If it rains, the club will lose the investment: $ -1,000

If it is sunny, the club will collect $4,500 ;

From all this the profit they will get

profit: 4,500 - 1000 = 3,500

And

There is a chance of 20% then

The club will loss the investment is

-1000*20 = -200

And

Effect on their profit if there is a rain then

3500*80 = 2800

then

The expected value of tournament is 2800-200=2600.

So,The expected value of tournament is $2600.

Learn more about Expected value here https://brainly.com/question/15858152

#SPJ4

Answer:

C

Step-by-step explanation:

on edge

please help with this

Answers

Answer:

a) x = 1, x = -7

b) (-3, -48)

c) x = -3

d) upwards

Step-by-step explanation:

Given quadratic equation:

[tex]y=3(x-1)(x+7)[/tex]

Part (a)

Intercept form of a quadratic equation:  

 [tex]y=a(x-p)(x-q)[/tex]

where:

p and q are the x-interceptsa is some constant

Comparing the formula with the given equation:

⇒ p = 1

⇒ q = -7

Therefore, the x-intercepts of the given equation are x = 1 and x = -7.

Part (b)

The midpoint between the two x-intercepts is the x-coordinate of the vertex.

[tex]\implies \textsf{Midpoint}=\dfrac{x_2+x_1}{2}=\dfrac{1+(-7)}{2}=-3[/tex]

Therefore, the x coordinate of the vertex is -3.

To find the y-coordinate of the vertex, substitute this into the given equation:

[tex]\implies y=3(-3-1)(-3+7)=-48[/tex]

Therefore, the coordinates of the vertex are (-3, -48).

Part (c)

The x-coordinate of the vertex is the axis of symmetry.

Therefore, the axis of symmetry is x = -3.

Part (d)

If the leading coefficient of a quadratic equation is positive, the parabola opens upwards.

If the leading coefficient of a quadratic equation is negative, the parabola opens downwards.

From inspection of the given equation, the leading coefficient is 3.

Therefore, the parabola opens upwards.

Learn more about quadratic equations here:

https://brainly.com/question/27997764

The sum of a positive number and 56 times its reciprocal is equal to its cube. What is the number?

Answers

Answer:

2×sqrt(2) = 2.828427125...

Step-by-step explanation:

x = "a positive number"

x + 56/x = x³

x² + 56 = x⁴

x⁴ - x² - 56 = 0

(x² + 7)(x² - 8) = 0

this is true only for

x² = -7

or

x² = 8

x² = -7 would mean that x is an imaginary number. so, that is not a valid solution here.

x² = 8 means

x = sqrt(8) = sqrt(4×2) = 2×sqrt(2) = 2.828427125...

Kim's softball team was playing in the championship game. When there were 444 innings left, the team was losing by a score of 171717 to 666 runs. In the last 444 innings, her team scored the same number of runs per inning, and the other team did not score any more runs. Kim's team won with the most runs.
Write an inequality to determine the number of runs per inning, ppp, Kim's team could have scored.
Find the minimum whole number of runs per inning Kim's team could have scored.

Answers

The inequality to determine the number of runs per inning, p Kim's team could have scored is; 4r + 6 > 17

How to write an Inequality?

Let r represent the number of runs per inning. Thus for 4 innings, we have 4r.

The team already has 6 runs. Now add the additional runs to this to get;

4r + 6

The team wants to score more than the other team, this means they need more than 17 and so the inequality required is;

4r + 6 > 17

Subtract 6 from each side to get;

4r + 6 - 6 > 17 - 6

4r > 11

Divide both sides by 4 to get:

r > 2.75

Approximating to a whole number gives;

r > 3

Read more about writing inequalities at; https://brainly.com/question/25275758

#SPJ1

Answer:

6+4p> 17
3

Step-by-step explanation:

9x/4y=1 and y= 18, what is the value of x​

Answers

Answer:

x = 8

Step-by-step explanation:

[tex]\frac{9x}{4y}=1[/tex]

[tex]\frac{9x}{4*18} =1[/tex]

9x = 72

x = 8

You select one red marble from the full bag in exercise 20. what is the probability that the next marble you select will be green without replacement of the first marble?

Answers

P(blue) = 1/3  

Step-by-step explanation:

Probability for any event A is given by

p(A) = no of times of happening of event A/ total no of occurrence of all the events given under the situation.

A bag contains 5 blue marbles, 6 red marbles, and 4 green marbles

So, total no of occurrences of all the events given under the situation

one can either pick blue, red, or 4 green marbles

Total no. of occurrence of all the event = 5 + 6 + 4 = 15

Now, no occurrence of selecting blue.

since there are 5 blue marbles, there is a chance of selecting blue as 5.

no occurrence of selecting blue = 5

Therefore,

P(blue) = no of occurrence of selecting blue /total no of occurrence of picking any color ball

P(blue) = 5/15 = 1/3.

Learn more about probability at

https://brainly.com/question/24756209

#SPJ4

A dindercal container close at boat and have a radius 7cm and hight 6cm find total surface area of the container what is the volume of this container

Answers

A) The total surface area of the given cylindrical container is 571.76 cm² and B) The volume of the given cylindrical container is 923.62 cm³.

What are the formulae for the total surface and volume of the cylinder?

Consider a cylinder with height 'h', and radius 'r'.

The total surface area of the cylinder(TSA) = 2πr (r + h) square units

and

The volume of the cylinder V = πr²h cubic units.

Calculation:

It is given that,

A cylindrical container closed on both ends has a radius r = 7 cm and a height h = 6 cm.

So,

TSA = 2πr (r + h)

       = 2× π × 7 × (7 + 6)

       = 14 × π × 13

      = 571.76 cm²

and

Volume = πr²h

             = π × 7² × 6

             = π × 49 × 6

             = 923.62 cm³

Therefore, the total surface area and volume of the cylindrical container are 571.76 cm² and 923.62 cm³ respectively.

Learn more about the cylinder and its surface area and volume here:

https://brainly.com/question/27795300

#SPJ4

What criteria might CBR researchers use to select and specify which units will be sampled as well as the sampling frame and methods to be used

Answers

The criteria which CBR researchers might use to select and specify which units will be sampled as well as the sampling frame and methods to be used is the population.

What is Population?

This is defined as the total number of organisms found in a particular place at any given point in time and varies based on the number of variables present. In some cases, head counts are done to ascertain the exact value of individuals present.

The probability sampling method is ideal for a large population and involves random selection of the organisms so as to find out the statistical features of the group being used such as the range, mean etc. This tool is very important in the area of giving estimates for  ascertain type of activity or job.

Read more about Population here https://brainly.com/question/25896797

#SPJ1

Jamie and owen joined a monthly walking challenge at their youth center. jamie has already walked 120 minutes and plans to walk 20 minutes per day. owen plans to walk 30 minutes per day. let x represent the number of days and y represent the total number of minutes walked during the challenge. which system of equations can be used to find the number of days it will take for owen and jamie to have walked the same number of minutes? a system of equations. y equals 20 x plus 120. y equals 30 x. a system of equations. y equals 20 x. y equals 30 x plus 120. a system of equations. y equals 120 x plus 20. y equals 30 x. a system of equations. y equals 120 x. y equals 30 x plus 20.

Answers

Answer:

The system of equation y=120+20x and y=30x

Step-by-step explanation:

Let the number of days be x and y be the total number minutes

Jamie has walked 120 mins. She will walk at a rate of 20 mins per day.

y=120+20x

Owen walks 30 mins a day. So

y = 30x

To learn more about Linear Equations visit https://brainly.com/question/1884491

#SPJ4

Find the indicated term of the geometric sequence.
5th term of 1,
1 /4,1/16,…

Answers

Answer:

[tex]\frac{1}{256}[/tex]

Step-by-step explanation:

So generally a geometric sequence can be defined as: [tex]a_n=a_1(r)^{n-1}[/tex] which is the explicit form. The r, is what each previous term is being multiplied by to get the next value which is evident in the recursive form: [tex]a_n = r(a_{n-1})\\[/tex]. Knowing this we can take two values which are "next" to each other to find what r is. In this case I'll just is 1 and 1/4, given these two values we know that: [tex]\frac{1}{4} = 1 * r[/tex], 1*r is just r... so what each term is being multiplied by is 1/4. So let's plug the values into the explicit formula: [tex]a_n=(\frac{1}{4})^{n-1}[/tex] (I didn't put an a_1 value in front, since it's just 1... so it's a bit redundant). Anyways using this formula we simply plug in 5 as n into the equation to find the 5th term: [tex]a_5 = (\frac{1}{4})^{5-1} = (\frac{1}{4})^4 = \frac{1^4}{4^4} = \frac{1}{256}[/tex]

When multiplying or dividing measured quantities, what determines the number of significant figures in the result?.

Answers

The quantity with the fewest number of significant figures.

The least number of significant figures is used to calculate the product's or quotient's number of significant figures when multiplying or dividing measurable numbers. While 5.25 has three major figures, 3.5 only has two.

What is significant figures?

Significant figures are the number of digits in a value, often a measurement, that contribute to the degree of accuracy of the value. We start counting significant figures at the first non-zero digit. Calculate the number of significant figures for an assortment of numbers.

When multiplying two measured values the number of significant figures should be equal to the?

For multiplication or division, the rule is to count the number of significant figures in each number being multiplied or divided and then limit the significant figures in the answer to the lowest count. An example is as follows: The final answer, limited to four significant figures, is 4,094.

Learn more about significant figures

brainly.com/question/14804345

#SPJ4

Can someone help with this problem please.
A sheet of paper, 12 inches by 18 inches, is folded so that 2 opposite corners touch, as shown in the figures below. What is the area, in square inches, of the shaded triangle formed as the result of the overlap. (Please look at the picture!)

Answers

The area of the shaded triangle formed as the result of the overlap is = 62.35 inches ²

Calculation of the equilateral triangle

After folding the rectangle with length of 12 inches and width of 18 inches, an equilateral triangle was formed.

An equilateral triangle is a type of triangle where by all the three sides are equal.

To determine the value of one of the sides, CB or CD is used because the folding didn't affect these sides.

Using the formula for the area of an equilateral triangle,

A = √¾ a²

a= 12 inches

A = √¾ ×12²

A = 62.35 inches ²

Learn more about triangle here:

https://brainly.com/question/1058720

#SPJ1

Answer:

Its 78!!!!

Solution:

1/2(12 * 18 - 5 * 12) = 78

David drives from his home to the airport to catch a flight. He drives 35 miles in the first hour, but realizes that he will be 1 hour late if he continues at this speed. He increases his speed by 15 miles per hour for the rest of the way to the airport and arrives 30 minutes early. How many miles is the airport from his home

Answers

The distance between David's home and the airport is 210 miles.

Distance traveled after 1 hour

let the distance traveled after first 1 hour = y

distance/speed = time

after 1 hour his speed = (35 + 15) = 50 mph

total time after 1 hour and 30 mins = 1.5 hr

y/50 + 1.5 = y/35

multiply through by350

7y + 525 = 10y

3y = 525

y = 525/3

y = 175 miles

In the first 1 hour, at 35 mph, distance covered = 35 miles

Distance from his home to airport = 175 miles + 35 miles = 210 miles

Thus, the distance between David's home and the airport is 210 miles.

Learn more about distance here: https://brainly.com/question/2854969

#SPJ1

Which equations are true for x = –2 and x = 2? Select two options x2 – 4 = 0 x2 = –4 3x2 + 12 = 0 4x2 = 16 2(x – 2)2 = 0

Answers

The equations which are true for the values of x given are; x²-4 =0 and 4x²= 16.

Which equations are true for the values of x?

The values of x given according to the task content are; x = –2 and x = 2.

By considering equation; x²-4 =0

x² = 4

x = ±2

Also, by considering 4x²= 16;

x² = 16/4 = 4

x = ±4.

Read more on square root;

https://brainly.com/question/3617398

#SPJ1

Other Questions
What is the midpoint of 125 and 12? Which requires more work, increasing a car's speed from 0 mph to 30 mph or from 50 mph to 60 mph? Identify which property you would use first to solve the following equation.3(2y+5) = 8y - 23 Dom is an editor for a small-town newspaper. He decides that he doesn't want to publish commentary about terrorism in the editorial section of the paper. He automatically tosses those he receives from other journalists and community members. Dom is performing the role of a _______ for the paper. The context of an unknown word might be described as all of the following, except___affixes within the wordwords surrounding the wordparagraphs surrounding the wordsentences surrounding the word Difference of Squares gives which complex factors for the expression x +11?A. (x+11)2(x-i11)B. (x+11)(x+11)C. (x-i11)(x-11)D. (x+11)(x - 11) How should the integral in gauss's law be evaluated? In general how did jeweish and christian interpreters interact with aristotelian cosmology? A nurse is educating a client who is prescribed the oral medication cyclosporine (Sandimmune) for rheumatoid arthritis. Which of the following information should the nurse include Though asian americans comprise only about 5.6 percent of the u.s. population, they represent? Where are phospholipids most likely found in a eukaryotic cell? group of answer choices ribosomes around organelles plasma membrane flagella b and c Which of the following statements holds true regarding sleep During IEP, the effective date is either the first of the month the member becomes eligible OR the first of the month following the month the enrollment form is submitted, whichever is later Storing foods out of sight and avoiding the path by the vending machines are examples of what behavior modification technique? pls help i need this ASAP If an issuing firm sells securities to the investing public without involving an investment banker, the issue is called:________ The body of a speech consists of a. thesis statements and introductions. b. a restatement of the thesis and statement of closure. c. specific purpose statements and internal summaries. d. main points, supporting points, and transitions. The value of a companys stock is represented by the expression x2 2y and the companys purchases are modeled by 2x 5y. the companys goal is to maintain a stock value of at least $5,000, while keeping the purchases below $1,000. which system of inequalities represents this scenario? x2 2y > 5000 2x 5y < 1000 x2 2y > 5000 2x 5y 1000 x2 2y 5000 2x 5y < 1000 x2 2y 5000 2x 5y 1000 1. Factor and simplify: cos xcsc x-cosxcot x 5 Jack works as a part-time waiter. He earns $60 per weekday and $98 per day on weekends. (a) On a particular week, he works from Tuesday to Sunday. How much was he paid for that week? (b) Find his average wage rate per day.